Math, asked by Anonymous, 5 months ago

Lt. sec x-2
x-π/3. x-π/3 Is equal to​

Answers

Answered by MaheswariS
1

\underline{\textbf{Given:}}

\mathsf{\displaystyle\lim_{x\to\;\frac{\pi}{3}}\;\dfrac{sec\,x-2}{x-\dfrac{\pi}{3}}}

\underline{\textbf{To find:}}

\textsf{The value of}

\mathsf{\displaystyle\lim_{x\to\;\frac{\pi}{3}}\;\dfrac{sec\,x-2}{x-\dfrac{\pi}{3}}}

\underline{\textbf{Solution:}}

\mathsf{Consider,}

\mathsf{\displaystyle\lim_{x\to\;\frac{\pi}{3}}\;\dfrac{sec\,x-2}{x-\dfrac{\pi}{3}}}

\mathsf{=\dfrac{sec\,\dfrac{\pi}{3}-2}{\dfrac{\pi}{3}-\dfrac{\pi}{3}}}

\mathsf{=\dfrac{0}{0}\;form}

\textsf{Apply L Hopital's rule,}

\mathsf{=\displaystyle\lim_{x\to\;\frac{\pi}{3}}\;\dfrac{sec\,x\;tan\,x}{1-0}}

\mathsf{=sec\dfrac{\pi}{3}\;tan\dfrac{\pi}{3}}

\mathsf{=2\,\sqrt{3}}

\implies\boxed{\mathsf{\displaystyle\lim_{x\to\;\frac{\pi}{3}}\;\dfrac{sec\,x-2}{x-\dfrac{\pi}{3}}=2\,\sqrt{3}}}

\underline{\textbf{Find more:}}

Lim. 8x³-1 / 16x4-1

x--->1/2

https://brainly.in/question/6423235  

If f(2)=4 and f'(2)=1, then find lim x tends to 2 xf(2)-2f(x)/x-2

https://brainly.in/question/5808208

Similar questions